0% found this document useful (0 votes)
277 views17 pages

2024_OMMC_MOCK_AMC_12_SOLUTIONS (2)

Uploaded by

ayushagrules
Copyright
© © All Rights Reserved
We take content rights seriously. If you suspect this is your content, claim it here.
Available Formats
Download as PDF, TXT or read online on Scribd
0% found this document useful (0 votes)
277 views17 pages

2024_OMMC_MOCK_AMC_12_SOLUTIONS (2)

Uploaded by

ayushagrules
Copyright
© © All Rights Reserved
We take content rights seriously. If you suspect this is your content, claim it here.
Available Formats
Download as PDF, TXT or read online on Scribd
You are on page 1/ 17

OMMC 2024 MOCK AMC 12 Solutions

The OMMC Staff

November 2024
SOLUTIONS

1. What is the value of


22 + 20
?
22 + 24

1 3 5 1 1
(A) (B) (C) (D) (E)
4 10 16 3 2

1
Answer: (A) 4

4+1 5 1
Solution: The fraction simplifies to 4+16
= 20 = (A)
4

2. Elise buys 1 raffle ticket for her school’s fundraiser. To determine the winner of the raffle,
a ticket is randomly selected among all raffle tickets purchased, and Elise wins the raffle
if her ticket is selected. If Elise had instead bought 100 raffle tickets, she would be 82
times more likely to win the raffle. What are Elise’s chances of winning the raffle if she
buys only 1 raffle ticket?
1 1 1 1 1
(A) (B) (C) (D) (E)
492 451 428 420 361

1
Answer: (B) 451
Solution: Let x be the number of raffle tickets other than Elise’s. Then we have that
1 100
· 82 =
x +1 x + 100
=⇒ 41(x + 100) = 50(x + 1)
=⇒ 9x = 4050
=⇒ x = 450.

1 1
The answer is therefore x+1 = (B) .
451
3. Saul has a sock drawer with socks that are either black or white, and either long or short.
One day, while reorganizing his drawer, Saul found that:

• His white socks and his black socks are in ratio 4 : 5;


• His long white socks and his long black socks are in ratio 2 : 3;
• His short white socks and his short black socks are in ratio 4 : 3.

What is the ratio of Saul’s long socks to his short socks?

(A) 3 : 1 (B) 5 : 4 (C) 13 : 8 (D) 18 : 5 (E) 20 : 7

Answer: (E) 20 : 7
Solution: Suppose that Saul has 2x long white socks, 3x long black socks, 4y short white
socks, and 3y short black socks. Then the ratio of his white socks to his black socks is
4 2x+4y
5
= 3x+3y , so 4(3x + 3y) = 5(2x + 4y) → 8y = 2x → x = 4y. Then the ratio of Saul’s long
5x 20y
socks to his short socks is 7y
= 7y
= (E) 20 : 7

4. Aarav, Brian, and Corey are standing in a line on an empty field so that Aarav is 133
meters away from Brian and 397 meters away from Corey. Brian and Corey are told to
clap immediately after hearing Aarav clap once. After Aarav claps once, he hears two
claps from Brian and Corey, one 1.6 seconds after the other.
Which of the following is closest to the speed at which sound traveled during this
experiment, in meters per second?

(A) 330 (B) 335 (C) 340 (D) 345 (E) 350

Answer: (A) 330


Solution: After Aarav claps once, he hears the clap from Brian after sound travels
133 + 133 = 266 meters and hears the clap from Corey after sound travels 397 + 397 = 794
meters. Therefore sound traveled 794 − 266 = 528 meters in 1.6 seconds, which is a speed
of 528
1.6 = (A) 330 meters per second.

5. Rosemary has a peculiar analog clock that chimes when its hour and minute hands are
perpendicular to each other. Her clock currently displays the time 7:00 PM. The next
time her clock chimes occurs between x and (x + 1) minutes from now, where x is a
positive integer. What is x?

(A) 5 (B) 6 (C) 16 (D) 21 (E) 22


Answer: (D) 21
Solution: Notice that every minute, the minute hand travels 6 degrees clockwise. The
hour hand makes one revolution in the time the minute hand makes 12, so it is 12 times
slower, traveling at 0.5 degrees clockwise per minute. This means that the angle between
the two hands changes by 5.5 degrees every minute. At 7 pm, the hour hand is 210
degrees clockwise from the minute hand. Therefore the amount of minutes required for
the hour hand to be 90 degrees clockwise from the minute hand is 210−90
11/2
= 240
11
9
= 21 + 11
which means x = (D) 21 .

6. What is the remainder when (2024!)


..
(5!) .
(4!)
(3!)
(2!)
is divided by 10?

(A) 0 (B) 2 (C) 4 (D) 6 (E) 8

Answer: (D) 6
Solution: Clearly the given quantity is even. Now observe that
(2024!)
..
(5!) .
(4!)
S = (3!)

is a multiple of 4 as 3! is even. Therefore by Fermat’s Little Theorem 2S ≡ 1 (mod 5). Then


the Chinese Remainder Theorem implies the answer is (D) 6 .

7. Let x and y be positive integers such that y is a factor of 143x , and y has exactly x 2 + 143
positive factors. What is the sum of the digits of the sum of all possible values of x?

(A) 6 (B) 7 (C) 8 (D) 9 (E) 10

Answer: (B) 7
Solution: We must have that y = 11p · 13q where p, q ≤ x. Then y has (p + 1)(q + 1)
factors. If both p, q < x then (p + 1)(q + 1) ≤ x 2 < x 2 + 143. If p < x − 1 or q < x − 1 then
(p + 1)(q + 1) ≤ (x − 1)(x + 1) = x 2 − 1 < x 2 + 143. Therefore p, q is x − 1, x in some order or
x, x. In the first case we have that x 2 + x = x 2 + 143 → x = 143. In the second case we have
that x 2 + 2x + 1 = x 2 + 143 → x = 71. The sum of these possibilities is 214 and the sum of
the digits is (A) 7 .
8. Let P be the (unique) monic
p p such that P has integer
polynomial of minimum degree
3 3
coefficients and P (1 + 5) = 0. What is the value of P (1 − 5)?

(A) − 10 (B) − 5 (C) 0 (D) 5 (E) 10

Answer: (A) −10


3
p 3 3
Solution:
p Let x = 1 + 5. Then notice that (x − 1) = 5, so P (x) = (x − 1) − 5. Then
3
P (1 − 5) = −5 − 5 = (A) − 10

9. Ana has a four digit number, and assigns each digit to one of the variables a, b, c, or d .
She notices that no matter how she assigns the digits to a, b, c, and d , the value of
ab + bc + cd + d a is always even. Given that Ana’s friend Bailey now selects a four digit
number at random, what is the probability his number also satisfies this property?
1 7 5 13 3
(A) (B) (C) (D) (E)
2 12 8 18 4

5
Answer: (C) 8
Solution: We can first factor ab + bc + cd + d a to a(b + d ) + c(b + d ) = (a + c)(b + d ).
Notice that if exactly two of a, b, c, d are odd, and the other two digits are even, then
(odd + even)(odd + even) = odd, which doesn’t work. In any other case, we are
guaranteed to have either odd + odd or even + even as one of the two factors, causing
(a + c)(b + d ) to be guaranteed to be even.
Now, we can do complementary counting. There are 4 even values of a and five odd
values of a. The other three variables have 5 possible even or odd values. The number of
possibilities with a even and one of b, c, and d even is 3 · 5 · 53 , and the number of
possibilities with a odd and one of b, c, and d odd is 3 · 4 · 53 . Therefore, the probability
that two of a, b, c, d are even, and the other two are odd is

3 · 5 · 53 + 3 · 4 · 53 3
= .
9000 8

5
So, our answer is 1 − 38 = (C) .
8

10. Suppose z is a complex number and x and y are nonzero reals such that

|z| = |z + x| = |z + yi | = 1.

What is x 2 + y 2 ?
p
2 p p
(A) (B) 2 (C) 2 (D) 2 2 (E) 4
2
Answer: (E) 4
Solution: Notice that the three points z, z + x, z + yi form a right triangle in the complex
plane. These points all lie on the unit circle, so the triangle has hypotenuse 2. Then

x 2 + y 2 = |x − yi |2 = |(z + x) − (z + yi )|2 = (E) 4 .

11. Convex quadrilateral ABC D has area 100. Point E lies on AB , and points F and G are the
midpoints of C E and DE , respectively. If the area of quadrilateral AF GB is 40, what is the
area of △E F G?

(A) 5 (B) 10 (C) 15 (D) 20 (E) 25

Answer: (B) 10
Solution: Let [A] denote the area of shape A. Then let [E F G] = x so that
[BC E ] + [ADE ] = 100 − x. Then observe that
x 100 − x
40 = [AGF B ] = [E F G] + [AGE ] + [B F E ] = + .
4 2
x
Solving gives x = 40, so [E F G] = 4 = (B) 10 .

12. Let a be a real number and a ̸= 1. What is the minimum possible integral value of
¯ ¯
¯a + 10 ¯?
¯ ¯
¯ a −1¯

(A) 5 (B) 6 (C) 7 (D) 8 (E) 9

Answer: (B) 6
10 10
¡ ¢
Solution: Notice that a + a−1 = 1 + (a − 1) + a−1 . By AM-GM, the minimum possible
p
absolute value of the second partp is 2 10. Then by the triangle inequality, the minimum
10
possible value of |a + a−1 | is 2 10 − 1 which has ceiling (B) 6
13. The side length of the larger tiles is denoted as x, and the side length of the smaller tiles
is denoted as y. The dotted lines form a 30◦ angle. What is the ratio xy ?

p p p p
(A) 2 (B) 2 3 (C) 2 + 3 (D) 3 2 (E) 3 + 2

p
Answer: (C) 2 + 3
Solution:

A B

C D

Label points A, B , C , and D on the diagram. Then, ∠D AC = 45◦ and ∠D AB = 30◦ .


Therefore, ∠C AB = 45◦ + 30◦ = 75◦ , so

p1 + 1 p
x ◦ ◦ ◦ 3
= tan 75 = tan(30 + 45 ) = = (C) 2 + 3.
y 1 − p1
3
14. Let the sequence (an ) satisfy a1 = 5 and

a1 + a2 + . . . + an = n 2 an

for all integers n > 1. What is a 25 ?


1 1 1
(A) (B) (C) (D) 625 (E) 1625
625 120 65

1
Answer: (C) 65
Solution 1: Write the equations for n = m and n = m + 1.

a1 + a2 + · · · + am = m 2 am
a 2 + a 2 + · · · + a m + a m+1 = (m + 1)2 a m+1 .

Subtracting the first equation from the second equation yields


m
a m+1 = (m + 1)2 a m+1 − m 2 a m and then rearranging yields a m+1 = m+2 a m . Therefore,

1 2 23 24
a 25 = 5 × × ×···× × .
3 4 25 26

5·1·2 1
This telescopes into the answer, 25·26 = (C) .
65
10
Solution 2: We claim that an = n(n+1) for all n ≥ 1. We will prove this by induction on n.
Base Case: n = 1
10
Then, a 1 = 5 = 1(2) .
Inductive Step:
Using the equation, we need to show
n 10 10
= n2
X
.
i =1 i (i + 1) n(n + 1)

10
Indeed, i (i +1) = 10 10
i − i +1 so the left hand side telescopes into

n 10
X 10 10 10n
− = 10 − = ,
i =1 i i +1 n +1 n +1

which is equal to the right hand side.

10 1
Therefore, we get that the answer is 25·26 = (C) .
65
15. 12 distinct mugs are each painted either red, white, blue, or yellow. Let n be the number
of ways to paint these mugs such that an odd number them are painted blue and an even
number of them are painted yellow. What is the remainder when n is divided by 5?

(A) 0 (B) 1 (C) 2 (D) 3 (E) 4

Answer: (E) 4
Solution: Consider all 412 possible ways to paint the mugs. Notice that half of them have
an odd total number of yellow and blue mugs, as after 11 mugs are painted half the
possibilities for the twelfth mug make the parity correct. In addition, out of the
remaining possibilities, by symmetry half of them will have an odd number of blue mugs
12
and an even number of yellow mugs. Therefore n = 44 = 411 , and modulo 5 this is
equivalent to (−1)11 ≡ −1 ≡ (E) 4 .

16. On rectangle ABC D with area 1, the midpoints of AB and BC are M and N , respectively.
Point X is chosen on segment AD and the intersections of M X and N X with AC are
points Y and Z , respectively. If AC = 4Y Z , what is the area of triangle X Y Z ?
1 1 1 1 1
(A) (B) (C) (D) (E)
18 16 12 9 8

1
Answer: (B) 16
Solution:

D C

Z
X N

A M B

Notice that M N ∥ AC , so M N ∥ Y Z . Since AC = 4Y Z and AC = 2M N , we have that


M N = 2Y Z . This tells us that for the similar triangles X Y Z and X M N , triangle X M N
has side lengths precisely twice as large as that of X Y Z . So, we get X Y = Y M and
X Z = Z N . Because of similar triangles Z AX and ZC N , we can see that X Z = Z N only
when AX = C N , meaning X must be the midpoint of AD. Now, it’s not hard to see that
X Y = Y M holds as well, since Y must now be the center of rectangle AM Z X . Thus,
1
[X Y Z ] = 41 · [AM Z X ] = 41 · 14 · [ABC D] = (B) .
16
17. In the given cone, the diameter X Y of the base circle is 2 units.
p Point Z is positioned
directly above the center O of the base circle, such that ZO = 2. A point W lies on the
X W to W
circle such that the ratio of the arc lengths Ú Ú Y is 2:1. What is the shortest
distance between X and the line Z W ?

1 3 p p
(A) (B) (C) 3 (D) 2 (E) 2 2
2 2

3
Answer: (B) 2
p
Solution: Note that X Z = Z W = W X = 3. Hence the distance from X to Z W is just the
p 3
height of an equilateral triangle with side length 3, which is (B) .
2

18. Six distinct positive integers from 1 to 35 are chosen uniformly and independently at
random. Given that 1 is one of these six integers, and it is possible to place these integers
on the 6 vertices of a regular hexagon such that the 3 pairs of opposite vertices have the
same sum, what is the expected value of this common sum?
51 53 55 57 59
(A) (B) (C) (D) (E)
2 2 2 2 2
57
Answer: (D) 2
Solution: Let the numbers be a < b < c < d < e < f . If we can pair them up to have the
same sum, these pairs must be a + f , b + e, and c + d .
Let the common sum be x. Since the smallest integer is 1, the largest integer must be
f = x − 1. We must count the number of ways to choose b, c, d , e such that
b + e = c + d = x and 1 < b < c < d < e < x − 1. The¥ number of unordered pairs of distinct
x−3
¦
integers 1 < m < n < x − 1 such that m + n = x is 2 , and (b, e) and (c, d ) correspond to
Ã¥ ¦!
x−3
two of these pairs, so we can choose the values of b, c, d , e in 2
ways. Note the
2
maximum value of x is 36. We can then compute the expected value as
Ã¥ ¦! Ã !
36 i −3 16
X
2
X i
i (4i + 7)
i =7 2 i =2 2
Ã¥ ¦! = Ã ! ,
36 i −3 16
X
2
X i
2
i =7 2 i =2 2

where the second sum can be derived à ! by Ãcombining


! Ã terms
! with the same combination in
i i +1 i
the initial sum. Note that (4i + 7) = 12 +3 , so the numerator is
2 3 2
à à ! à !! à à ! à !! à ! à !
16
X i +1 i X16 i +1 i 18 17
12 +3 = 12 +3 = 12 +3
i =2 3 2 i =2 3 2 4 3

à !
17
due to Hockey Stick. Also using Hockey Stick, the denominator is 2 . Therefore, the
3
answer is à ! à !
18 17
12 +3
4 3 57
à ! = (D) .
17 2
2
3
p
3 p p3 p
19. The sum of all real numbers a for which 2 + a + 2 − a is an integer can be written
p
as q , where p and q are relatively prime positive integers. What is p + q?

(A) 6 (B) 127 (C) 135 (D) 262 (E) 289

Answer: (D) 262


p
3 p p
3 p p
3
Solution: Set x = 2 + a, y = 2 − a and x + y = m, an integer. We know x y = 4 − a
and

4 = x3 + y 3
= (x + y)(x 2 − x y + y 2 )
p
= m 3 − 3m 4 − a
3

=⇒ (m 3 − 4)3 = 27m 3 (4 − a)
108m 3 − (m 3 − 4)3
=⇒ a = .
27m 3

We must have a > 0. Since x 3 + y 3 = (x + y)(x 2 − x y + y 2 ) > 0 and


¢2
x 2 − x y + y 2 = x − 12 y + 43 y 2 > 0, we must have x + y = m > 0. Therefore,
¡

108m 3 − (m 3 − 4)3 > 0, which simplifies to

m 9 − 12m 6 − 60m 3 − 64 = (m 3 − 16)(m 3 + 2)2 < 0.

Therefore, m 3 < 16, so m = 1 or m = 2.


If m = 2, then a = 100
27 , and if m = 1, then a = 5.
100
Therefore, the only possible values of a are 5 and 27
, so the sum is 5 + 100
27
= 235
27
, giving
the answer 235 + 27 = (D) 262 .

20. A line ℓ passing through vertex A of equilateral triangle ABC cuts through the triangle.
The distances from B and C to line ℓ are 10 and 7, respectively. What is BC 2 ?

(A) 282 (B) 292 (C) 302 (D) 312 (E) 322
Answer: (B) 292
Solution:

B M D C
F

Let D be the intersection of ℓ with BC , let E be the foot from B to ℓ, and let F be the foot
from C to ℓ. Since △B E D ∼ △C F D and CB EF = 10 BD 10
7 , we get DC = 7 . Now, let x = BC , so
10
B D = 17 x. Define M to be the midpoint of BC . Then, AM 2 = AB 2 − B M 2 = 34 x 2 , and
¢2
7
D M 2 = 21 x − 17 x = 3492 x 2 . Therefore,
¡

¶µ
2 2 3 2 9 3 · 292
AD = AM + D M = + 2 x2 = .
4 34 342
q
AD BD 10 3
Now, since △AM D ∼ △B E D, we get M D = E D , so E D = 17 292 x. Now, the Pythagorean
Theorem gives

100 2 100 2 3 100 2


100 = B E 2 = B D 2 − E D 2 = x − x = x ,
289 289 292 292

so x 2 = (B) 292 .

21. Let f (x) = x 3 + 11x − 5 have roots a, b, c. What is f (a + 1) f (b + 1) f (c + 1)?

(A) 1716 (B) 3402 (C) 6885 (D) 7155 (E) 9416
Answer: (D) 7155
Solution: We have

f (a + 1) = (a + 1)3 + 11(a + 1) − 5 = a 3 + 3a 2 + 14a + 7 = 3a 2 + 3a + 12.

Therefore,

f (a + 1) f (b + 1) f (c + 1) = 27(a 2 + a + 4)(b 2 + b + 4)(c 2 + c + 4).

Let r and s be the roots of x 2 + x + 4. Then, this product is

27(a −r )(a − s)(b −r )(b − s)(c −r )(c − s) = (r − a)(r −b)(r −c)(s − a)(s −b)(s −c) = f (r ) f (s).

We have
f (r ) = r 3 + 11r − 5 = (r 2 + r + 4)(r − 1) + 8r − 1 = 8r − 1.
Therefore,

f (r ) f (s) = (8r − 1)(8s − 1) = 64r s − 8(r + s) + 1 = 64(4) − 8(−1) + 1 = 265.

This implies that the expression is equal to 27 · 265 = (D) 7155 .

22. The volume of the region in the positive x y z-space defined by the equation
m
⌊x + y⌋ + ⌊y + z⌋ + ⌊z + x⌋ = 21 can be written as n , where m and n are relatively prime
positive integers. What is m + n?

(A) 17 (B) 59 (C) 70 (D) 89 (E) 125

Answer: (E) 125


Solution: Note that ⌊x⌋ + ⌊y⌋ − ⌊x + y⌋ is equal to 0 if {x} + {y} < 1 and is equal to 1 if
{x} + {y} ≥ 1. Let a i be the probability that exactly i of the inequalities {x} + {y} ≥ 1,
{y} + {z} ≥ 1, and {z} + {x} ≥ 1 are true. Then, this is equivalent to picking p, q, and r
uniformly at random from [0, 1] and counting the number of inequalities of p + q ≥ 1,
q + r ≥ 1, and r + p ≥ 1 are true. By swapping p, q, and r with 1 − p, 1 − q, and 1 − r , the
truth of each inequality is flipped, so a 0 = a 3 and a 1 = a 2 , which means a 2 + a 3 = 12 . Also,
the first two inequalities are true if and only if p, r ≥ 1 − q, so the probability this happens
is 13 because p, 1 − q, and r are distributed uniformly and independently in [0, 1]. This
means 31 a 2 + a 3 = 13 since there is a one third chance that when two inequalities are true,
those inequalities are the first two inequalities. Solving gives a 0 = a 1 = a 2 = a 3 = 14 .
Now, we must either have ⌊x⌋ + ⌊y⌋ + ⌊z⌋ = 9 and three of the inequalities are true or
⌊x⌋ ¡12¢+ ⌊z⌋ = 10 and one of the inequalities is true. By stars and bars, there are
¡11¢+ ⌊y⌋
2 + 2 = 121 values of ⌊x⌋, ⌊y⌋, and ⌊z⌋, and each combination of values corresponds
to a volume of 14 , which means the total volume is 1214
. Therefore,
m + n = 121 + 4 = (E) 125 .
23. What is the sum of the digits of the smallest positive integer n such that n n + 2n is
divisible by 1000?

(A) 5 (B) 6 (C) 7 (D) 8 (E) 9

Answer: (B) 6
Solution: Clearly, n ≥ 5. The number is divisible by 1000 if it is divisible by 8 and 125.
Since 8 | 2n , 8 | n n , so n is even. Since 5 | n n + 2n and 5 ∤ 2n , 5 ∤ n. Then, n cannot be a
multiple of 4 since otherwise n n + 2n ≡ 2 (mod 4). This implies

n ≡ 2 (mod 4).

Let n = 2k, where k is odd. Now, this is equivalent to

125 | (2k)2k + 22k ,

or
125 | k 2k + 1.
This implies 125 | k 4k − 1. Taking modulo 5 in the original equation gives 5 ∤ n, so 5 ∤ k.
Therefore, 125 | k 100 − 1 by Euler’s Theorem, so

125 | k gcd(4k,100) − 1 = k 4 − 1.

If 5 | k 2 − 1, then 5 | k 2k − 1 = (k 2k + 1) − 2, contradiction. Therefore, 125 | k 2 + 1. We claim


that if 125 | k 2 + 1, then n = 2k satisfies the equation. This is true because k 4 ≡ 1
(mod 125) and 2k ≡ 2 (mod 4), so k 2k + 1 ≡ k 2 + 1 ≡ 0 (mod 125).
Now, it suffices to find the smallest k such that 125 | k 2 + 1. There are exactly two possible
values of k which are negatives of each other modulo 125. Since 25 | 72 + 1, we get k ≡ ±7
(mod 25). Now, let k = 25m + 7. We get

k 2 + 1 ≡ 350m + 50 (mod 125),

so m = 2. Therefore, k ≡ ±57 (mod 125), so the minimum value of k is 57. This works
because 125 | 572 + 1 = 3250. This means that the minimum value of n is 2k = 114. The
sum of the digits is 1 + 1 + 4 = (B) 6 .
24. Given a positive integer n, let x n denote the number of ordered pairs of integers (a, b)
such that a n + b n is divisible by 109, where 0 ≤ a, b ≤ 108. What is x 1 + x 2 + · · · + x 108 ?

(A) 34884 (B) 34992 (C) 35100 (D) 35208 (E) 35316

Answer: (C) 35100


Solution: We will find the value by computing the total number of ordered triples
(a, b, n) with 0 ≤ a, b ≤ 108 and 1 ≤ n ≤ 108 such that a n + b n ≡ 0 (mod 109).
Firstly, notice that if a = 0, b = 0 is always a solution for any choice of n. Furthermore,
a = 0 ⇒ b = 0, and vice versa. This gives 108 ways.
¡ ¢n
Now if neither a nor b is 0, we have that a n ≡ b n (mod 109) ⇒ ba ≡ −1 (mod 109).
Notice that there are 108 ways to pick a, b given that ba ≡ x (mod 109) for a fixed x, as
every value of a between 1 and 108, inclusive, has a multiplicative inverse in Fp .
Hence, it suffices to compute the number of ways to pick x, n such that x n ≡ −1
(mod 109).
Fix the order of x modulo 109 to be some divisor d of 108. It is known that there are ϕ(d )
ways to pick such a number x. Notice that if d is odd, then there cannot exist an n such
that x n ≡ 1 (mod 109). However, if d is even, then n ≡ d2 (mod d ) ⇒ x n ≡ 1 (mod 109).
Hence, there are 108
d ways to pick n in this case.
Then it suffices to compute

X 108ϕ(d ) X ϕ(d )
= 108 .
2|d |108 d 2|d |108 d

ϕ(d )
Notice that d is multiplicative, so we can express the sum over the prime powers:

ϕ(2) ϕ(4) ϕ(1) ϕ(3) ϕ(9) ϕ(27)


µ ¶µ ¶
+ + + + .
2 4 1 3 9 27

This can be computed to be (C) 35100 .

25. An ant sits at (0, 0) on the Cartesian plane. Each minute, it marches either one unit up or
one unit right. Suppose there are n distinct paths the ant can take to (10, 10) that
intersect the line x = y at exactly 5 points (not including (0, 0) and (10, 10)). What is the
sum of the digits of n?

(A) 15 (B) 18 (C) 21 (D) 24 (E) 27


Answer: (D) 24
p
Solution 1: Consider the line segment from (0, 0) to (10, 10) with length 10 2. Selecting
5 points on this segment
p effectively partitions
p it into 6 subsegments, each with lengths
that are multiples of 2, summing to 10 p 2. We solve this using generating functions by
assigning each subsegment a length i 2, with the number of ways to traverse the
segment given by 2 ×C i −1 , where C 0 = 1. The generating function becomes:

26 (x + x 2 + 2x 3 + 5x 4 + 14x 5 )6

We seek the coefficient of x 10 , which simplifies to finding the coefficient of x 4 in

(1 + x + 2x 2 + 5x 3 + 14x 4 )6 .

This involves partitioning 4:


- x 4 : 61 · 14 = 84
¡¢

- x 3 · x: 61 · 5 · 51 = 150
¡¢ ¡¢

- x 2 · x 2 : 62 · 22 = 60
¡¢

- x 2 · x · x: 61 · 2 · 52 = 120
¡¢ ¡¢

- x · x · x · x: 64 = 15
¡¢

Adding these gives 429, so the final answer is 26 · 429 = 27456. The sum of the digits is
2 + 7 + 4 + 5 + 6 = (D) 24 .
Solution 2: Similarly to the first solution, we will count the number of paths that always
stay at or below the line y = x that intersect the line at exactly 5 points excluding the start
and end points, then multiply this count by 26 = 64. We will form a bijection between
these paths and paths that start at (0, 0) and end at (5, 10) which never cross y = x + 5.
Consider a sequence of ten U s and ten Rs corresponding to a possible path. Whenever
the ant reaches y = x, including at (0, 0), delete the R directly after, and delete the final U .
Then, exactly six Rs and one U are deleted, and the new sequence corresponds to a path
from (0, 0) to (4, 9). This operation can be reversed. Given a sequence of nine U s and four
Rs such that the path never crosses y = x + 5, we can get a path with ten U s and ten Rs
satisfying the conditions by adding a U at the end of the sequence, then inserting an R
directly after whenever the path touches y = x + i for the first time for 0 ≤ i ≤ 5, including
the point (0, 0) for i = 0. The new path intersects y = x precisely before the Rs were
inserted, so it intersects y = x at exactly five points excluding the start and end points.
Now, we will count the number ¡of ¢paths starting from (0, 0) and ending at (4, 9) which
never cross y = x + 5. There are 13 4 = 715 total paths. We will count the number of paths
that cross the line y = x + 5. Consider the point where the path first crosses y = x + 5, so
the point is on y = x + 6. Swap all moves after that point, so right becomes up and up
becomes right. This gives
¡ ¢ a bijection from paths that cross the line to all paths from (0, 0)
to (3, 10), so there are 13
3
= 286 paths that cross the line. Therefore, the number of paths
that do not cross y = x + 5 is 715 − 286 = 429. Now, the total number of paths for the
original question is 64 · 429 = 27456, so the answer is 2 + 7 + 4 + 5 + 6 = (D) 24 .

You might also like